Why is B correct and D wrong?
Could anyone please explain why D is wrong? Other people in previous threads have asked the same ...
filozinni on September 5, 2020
  • June 2019 LSAT
  • SEC1
  • Q15
2
Replies
Help
Can you please explain the correct answer
JayDee8732 on September 5, 2020
  • October 2001 LSAT
  • SEC1
  • Q4
2
Replies
Explanation?
I got this one correct but I don't feel that confident about it. Can someone please explain?
kelsgorman on September 4, 2020
  • December 2010 LSAT
  • SEC2
  • Q16
3
Replies
Doctor Yamata
I'm totally confused with this game - sorry for the next question! As I noted in my previous ques...
Anna20 on September 4, 2020
  • February 1993 LSAT
  • SEC2
  • Q11
1
Reply
Oct2000-S2-Q20
Why is E more correct than A?
maonuo on September 4, 2020
  • October 2000 LSAT
  • SEC2
  • Q20
1
Reply
Why is A wrong?
Why is A wrong? And why is D the better answer?
Veda-Bhadharla on September 4, 2020
  • October 2000 LSAT
  • SEC2
  • Q20
1
Reply
question 16
I don't quite understand why the main passage is A--->B, B--->A? Where can we draw B--->A from th...
kens on September 4, 2020
  • September 2016 LSAT
  • SEC4
  • Q16
3
Replies
Doctor Yamata
Why is the correct answer here not C? What is the process for solving this question? In my se...
Anna20 on September 3, 2020
  • February 1993 LSAT
  • SEC2
  • Q10
1
Reply
In / Out Grouping Games
Is there a full list of in / out grouping games similar to this? I definitely need more practice ...
Anna20 on September 3, 2020
  • September 2009 LSAT
  • SEC4
  • Q23
1
Reply
Video Explanation
I have a question regarding your video explanation. You sort the grades in order [GP]>E>H. Then y...
Aley on September 3, 2020
  • June 1992 LSAT
  • SEC2
  • Q6
1
Reply
Q7
I'm not clear with this question. Can you please help me understanding this Question in a better ...
Saksham-Sabarwal on September 3, 2020
  • June 2007 LSAT
  • SEC3
  • Q7
2
Replies
A vs. E
Why is answer choice A incorrect? Principle: If likely to fix practical problem, then obligat...
ikarus on September 3, 2020
  • June 2012 LSAT
  • SEC4
  • Q14
4
Replies
Explanation
Must introduces necessary condition, correct? I diagrammed the question as follows: GP: Both th...
Joey77 on September 3, 2020
  • June 2012 LSAT
  • SEC4
  • Q7
1
Reply
Game setup
Could you show the game setup here so I can compare how I started?
BrundonDavey on September 2, 2020
  • June 2016 LSAT
  • SEC2
  • Q6
2
Replies
Please Help!
@lsatmax I'm sort of confused between why answer choice (A) is correct and answer choice (C) is i...
Lauren-Au on September 2, 2020
  • June 2011 LSAT
  • SEC1
  • Q25
2
Replies
Explain
Please explain why A is correct rather than the other options
texasjohnrh on August 28, 2020
  • October 2010 LSAT
  • SEC5
  • Q20
2
Replies
Why not A?
I don't really understand the answer choice explanation that splitting gov't censorship and boyco...
lsatstudier1 on August 28, 2020
  • December 2009 LSAT
  • SEC4
  • Q10
1
Reply
Please diagram
I have no idea what I am missing here. Please diagram.
KGregory on August 28, 2020
  • June 1998 LSAT
  • SEC3
  • Q13
1
Reply
Why is B correct? Why is A incorrect?
Thanks
Ryan-Mahabir on August 27, 2020
  • October 2015 LSAT
  • SEC2
  • Q1
2
Replies
Explanation
I originally chose C and then I switched it to B. The reason was that I thought that the fact tha...
avif on August 27, 2020
  • December 1999 LSAT
  • SEC4
  • Q17
4
Replies